Convolution (Solved Problem 1)

Convolution (Solved Problem 1)

Neso Academy

6 лет назад

294,071 Просмотров

Ссылки и html тэги не поддерживаются


Комментарии:

Kritika Bhateja
Kritika Bhateja - 14.08.2023 20:19

Why are we not taking 3 cases , {- infinity,0}, {0, t}, {t , infinity}.
Also IF we are only taking 2 cases {- infinity,0}, {0, infinity}, then in solution its showing y(t) = 0, t<0 and y(t) = t , t>=0.
I didn't get where did t to infinity portion went ? If its 0 then why in solution we took , t>=0

Ответить
Hardik Jain
Hardik Jain - 07.08.2023 10:43

we can write the given func as unit signals and then use u*u as ramp and get the answer directly using this method saves a lot of effort :)

Ответить
C. Cody.
C. Cody. - 14.04.2023 07:17

professor when you convert from h(-𝜏) to h (t-𝜏). The curve should shift right instead of left. This is because the shifting operation is performed on the 𝜏 component only but no the whole (t-𝜏) argument. My professor taught us to let another function g(𝜏) = h(-𝜏), then perform subtraction or addition to g(𝜏) so that h(-𝜏) becomes h(t-𝜏), in this case we perform subtraction: g(𝜏-t) = h (-(𝜏-t)) = h(t-𝜏) (note that the shifting is performed only on 𝜏 but not the whole argument). Finally, because of g(𝜏-t), -t means that the whole curve shifts to the right by t units. Therefore it should shift to right instaead of left.

Ответить
CK_Wiings
CK_Wiings - 21.11.2022 17:35

These subject is very boring, embarrassing 🤯🤯 headache WTF

Ответить
Asma Ul Hosna
Asma Ul Hosna - 18.08.2022 04:55

In case II , t>0 range, why is the limit [-∞,∞]. Wouldn't it be [0,∞]? h(t) overlapped x(t) from 0.
confused about it. Hope you will help.

Ответить
yogi pro
yogi pro - 23.01.2022 19:28

Is answer y(t)=t.....for t>0.

Ответить
Sadat lucas
Sadat lucas - 02.12.2021 00:43

thanks a lot bro..

Ответить
ayush agarwal
ayush agarwal - 23.11.2021 21:13

method 2 op , thanks sir

Ответить
Krishna Yadav
Krishna Yadav - 07.01.2021 19:27

Great sir.....u cleared my confusion

Ответить
sedlyf sedlyf
sedlyf sedlyf - 18.11.2020 18:12

sir ne t negative li hai

Ответить
sai sunayan
sai sunayan - 28.09.2020 16:59

I think in method-1,h(-T) should be shifted towards right to get h(t-T)

Ответить
Green Emerald
Green Emerald - 14.09.2020 09:59

We can use trick # u(t+a) * u(t+b) = ramp(t+a+b) to solve question

Ответить
Abhay Kondru
Abhay Kondru - 10.09.2020 12:56

How invees Laplace of 1/s² is r(t) y not t

Ответить
Kunal Chouhan
Kunal Chouhan - 28.11.2019 20:43

sir convulated u(t-1)+u(t-5)-u(t-3)with ramp function plzzzz......

Ответить
BEERAM AISHWARYALAKSHMI
BEERAM AISHWARYALAKSHMI - 18.03.2019 04:19

Sir please method-1 confusion. I didn't get properly. Please explain me in a proper manner. From reversal I didn't get

Ответить
PRATIUSH ANAND
PRATIUSH ANAND - 11.03.2019 05:15

sir the limits of integration must be from minus infinity to 't'.

Ответить
Rahma Elsaeed 201501344
Rahma Elsaeed 201501344 - 05.12.2018 03:00

I watched previous lecture but I have not understand yet why shift isn't to right , could you explain it more ,please ?

Ответить
Shaik Afsana1610
Shaik Afsana1610 - 08.11.2018 12:43

good very helpfull

Ответить
Anonymous Person
Anonymous Person - 21.09.2018 18:07

Gate ques ans will be x(-t-t0)

Ответить
Akash Jagtap
Akash Jagtap - 30.08.2018 19:51

Sir why you take limits as -ilinfinity to +infinity
because signal overlaping is from '0' to 't'

Ответить
Jay Shah
Jay Shah - 02.05.2018 17:17

Shifting is done wrong because h(- τ+t) indicates folding and delay operation. So it is obtained by shifting h(- τ) towards right!!

Ответить
ZVirus007
ZVirus007 - 07.04.2018 18:01

Sir ,in case no.2 how the result of output as "t" was came by the integrating w.r.t tau from -inf to +int

Ответить
sai sumanth
sai sumanth - 24.03.2018 17:31

sirr take h(t)as ramp and one is exponential signal plzzz convulated

Ответить
Abdul Jabbar
Abdul Jabbar - 23.03.2018 23:01

why you not put the limites .

Ответить
Sandeep Chaudhary
Sandeep Chaudhary - 26.12.2017 17:31

Plzzz cover more gate question sir

Ответить
Dwayne Anthony
Dwayne Anthony - 25.10.2017 14:39

Great videos keep up the amazing work!

Ответить
Saikat Podder
Saikat Podder - 19.09.2017 08:23

Sir complete signal system as soon as possible

Ответить
ROHITH POKALA
ROHITH POKALA - 19.09.2017 06:58

Sir when will you start Fourier transforms?

Ответить